According to a survey, high school girls average 100 text messages daily (The Boston Globe, April 21, 2010). Assume the population standard deviation is 20 text messages. Suppose a random sample of 50 high school girls is taken. [You may find it useful to reference the z table. a. What is the probability that the sample mean is more than 105? (Round "z" value to 2 decimal places, and final answer to 4 decimal places.) Probability b. what is the probability that the sample mean is less than 95? (Round "z" value to 2 decimal places, and final answer to 4 decimal places.) Probability 0.0384 c. What is the probability that the sample mean is between 95 and 105? (Round "z" value to 2 decimal places, and final answer to 4 decimal places.) Probability 0.9232

Answers

Answer 1

The probability that the sample mean is more than 105 is 0.0384. The probability that the sample mean is less than 95 is 0.0384. The probability that the sample mean is between 95 and 105 is 0.9232.

The probability that the sample mean is more than 105 can be calculated using the following formula: P(X > 105) = P(Z > (105 - 100) / (20 / √50))

where:X is the sample mean

Z is the z-score

100 is the population mean

20 is the population standard deviation

50 is the sample size

Substituting these values into the formula, we get: P(X > 105) = P(Z > 1.77)

The z-table shows that the probability of a z-score greater than 1.77 is 0.0384. Therefore, the probability that the sample mean is more than 105 is 0.0384.

The probability that the sample mean is less than 95 can be calculated using the following formula: P(X < 95) = P(Z < (95 - 100) / (20 / √50))

Substituting these values into the formula, we get: P(X < 95) = P(Z < -1.77)

The z-table shows that the probability of a z-score less than -1.77 is 0.0384. Therefore, the probability that the sample mean is less than 95 is 0.0384.

The probability that the sample mean is between 95 and 105 can be calculated using the following formula: P(95 < X < 105) = P(Z < (105 - 100) / (20 / √50)) - P(Z < (95 - 100) / (20 / √50))

Substituting these values into the formula, we get: P(95 < X < 105) = P(Z < 1.77) - P(Z < -1.77)

The z-table shows that the probability of a z-score between 1.77 and -1.77 is 0.9232. Therefore, the probability that the sample mean is between 95 and 105 is 0.9232.

Visit here to learn more about standard deviation:

brainly.com/question/24298037

#SPJ11


Related Questions

Out of a random sample of 50 students at Lakeview community college, 30 were taking at least one course online. Compute D. Write in decimal form rounded to 2 decimal places.

Answers

To compute the proportion of students taking at least one course online, we divide the number of students taking at least one online course by the total sample size.

Proportion of students taking at least one online course = Number of students taking at least one online course / Total sample size.  In this case, the number of students taking at least one online course is given as 30, and the total sample size is 50. Proportion of students taking at least one online course = 30 / 50 = 0.60.

Therefore, the proportion of students taking at least one course online is 0.60, which can be written in decimal form as 0.60 (rounded to 2 decimal places).

To learn more about proportion click here: brainly.com/question/26974513

#SPJ11

Consider the 2x2 matrix À tè lor ) a. Determine the eigenvalues and the corresponding eigenvectors. B.Show that the eigenvectors are mutually perpendicular, C.Show that they satisfy the completeness relation, d.Find a unitary matrix which diagonalize A.

Answers

For the given 2x2 matrix A, we will determine the eigenvalues and corresponding eigenvectors. We will show that the eigenvectors are mutually perpendicular and satisfy the completeness relation. Finally, we will find a unitary matrix that diagonalizes A.

a) To find the eigenvalues and eigenvectors of matrix A, we solve the characteristic equation det(A - λI) = 0, where λ is the eigenvalue and I is the identity matrix. By solving the equation, we obtain the eigenvalues.
b) The corresponding eigenvectorscan be found by substituting the eigenvalues back into the equation (A - λI)x = 0 and solving for x. The resulting vectors are the eigenvectors.
c) To show that the eigenvectors are mutually perpendicular, we can check if their dot product is zero. If the dot product of two eigenvectors is zero, it indicates that they are orthogonal or mutually perpendicular.
d) The completeness relation states that the eigenvectors of a matrix form a complete set, meaning any vector in the space can be expressed as a linear combination of the eigenvectors.e) To diagonalize matrix A, we need to find a unitary matrix U such that U^(-1)AU = D, where D is a diagonal matrix. This can be achieved by setting the columns of U to be the normalized eigenvectors of A.
By following these steps, we can determine the eigenvalues and eigenvectors, show their orthogonality, verify the completeness relation, and find the unitary matrix that diagonalizes matrix A.

learn more about eigenvalues here

https://brainly.com/question/30357013



#SPJ11

A particle moves in a straight line with velocity v(t) = t^2 – 2t – 3 cm/s where t > 0 (a) Determine the point at which the particle has a constant velocity (b) After 2 seconds, the particle is located 3cm to the left of the origin. Determine s(t) (c) Calculate the total distance travelled by the particle in the first 5 seconds of motion

Answers

The particle has a constant velocity at t = 1s. The position function s(t) is s(t) = (t^3)/3 - t^2 - 3t + 7. The total distance travelled by the particle in the first 5 seconds of motion is approximately 11.67 cm.

(a) To determine the point at which the particle has a constant velocity, we need to find when its acceleration is equal to zero. This will allow us to locate the point at which the particle has a constant velocity. The derivative of the velocity function is what determines the acceleration, and it looks like this: a(t) = v'(t) = 2t - 2. After solving for t and setting this equal to zero, we see that t is equal to 1s.

(b) We need to integrate the velocity function in order to determine s(t), which is as follows: s(t) = ∫v(t)dt = (t^3)/3 - t^2 - 3t + C. To solve for C, we can make use of the starting condition that states that after two seconds, the particle will be situated three centimetres to the left of the origin. -3 = (2^3)/3 - 2^2 - 3*2 + C, so C = 7. Therefore, s(t) equals (t3)/3 minus t2 minus 3t plus 7.

(c) In order to determine the entire distance that the particle travelled in the first five seconds of its motion, we need to assess the difference between |s(5)| and |s(0)|, which is equal to |(53)/3 - 52 - 35 + 7 - (03)/3 + 02 + 30 - 7|, which is equal to 11.67 cm.

Learn more about function here:

https://brainly.com/question/30721594

#SPJ11

Find a matrix P, that diagonalize matrix A. Compute B = P-¹AP. Write clean, and clear. Show steps of calculations.
A = [9 -3 3]
[-3 6 -6]
[ 3 -6 6]

Answers

We are given matrix A and we need to find a matrix P that diagonalizes A. We will compute the matrix B = P⁻¹AP, where P is the matrix of eigenvectors of A.

This process involves finding the eigenvectors and eigenvalues of A, constructing P, and then computing B. We will show the step-by-step calculations. To diagonalize matrix A, we need to find a matrix P that consists of eigenvectors of A and compute the matrix B = P⁻¹AP. Let's go through the steps:

Step 1: Find the eigenvalues of matrix A:

To find the eigenvalues, we solve the characteristic equation det(A - λI) = 0, where λ is the eigenvalue and I am the identity matrix.

det(A - λI) = 0

|9-λ -3 3 |

|-3 6-λ -6|

| 3 -6 6-λ| = 0

Expanding the determinant and solving, we get the eigenvalues λ₁ = 0, λ₂ = 6, λ₃ = 15.

Step 2: Find the eigenvectors corresponding to each eigenvalue:

For each eigenvalue, we solve the equation (A - λI)X = 0, where X is the eigenvector.

For λ₁ = 0:

( A - 0I)X = 0

|9 -3 3 |

|-3 6 -6|

|3 -6 6 | X = 0

Solving this system, we find the eigenvector X₁ = [1 1 1].

For λ₂ = 6:

( A - 6I)X = 0

|3 -3 3 |

|-3 0 -6|

|3 -6 0 | X = 0

Solving this system, we find the eigenvector X₂ = [1 -2 1].

For λ₃ = 15:

( A - 15I)X = 0

|-6 -3 3 |

|-3 -9 -6|

|3 -6 -9| X = 0

Solving this system, we find the eigenvector X₃ = [-1 -2 1].

Step 3: Construct matrix P using the eigenvectors:

Matrix P is formed by placing the eigenvectors X₁, X₂, and X₃ as columns.

P = [1 1 -1]

[1 -2 -2]

[1 1 1]

Step 4: Compute matrix B = P⁻¹AP:

B = P⁻¹AP

B = P⁻¹(AP)

We compute P⁻¹ first:

P⁻¹ = (1/3) * [1 -1 0]

[0 1 -1]

[-1 1 1]

Then, we substitute the values into B = P⁻¹AP:

B = P⁻¹AP

B = (1/3) * [1 -1 0] * [9 -3 3]

[0 1 -1] [1 -2 1]

[-1 1 1] [1 1 1]

Multiplying the matrices, we get:

B = [6 0 0]

[0 0 0]

[0 0 15]

Learn more about eigenvectors here:- brainly.com/question/31043286

#SPJ11

Consider the following bounded variables linear program:

Maximize x_{1} + x_{2}
subject to - 2x_{1} + x_{2} <= 2
x_{1} - x_{2} <= 0
- 2 <= x_{1} <= 2
- 1 <= x_{2} <= 2

a. Solve the problem graphically in the (x_{1}, x_{2}) space.
b. Give all optimal basic feasible partitions. (Specify sets of basic and nonbasic variables at optimality.)
c. For the extreme point (x_{1}, x_{2}) = (0, 2) construct the bounded vari- ables simplex tableau and perform one iteration. Is the resulting tableau optimal?
d. Graphically verify whether the following is true or false. Starting at the point where the slack from the second constraint and x_{2} are nonbasic at their lower bounds, if one introduces x_{2} into the basis, then the resulting basic feasible solution is optimal.
e. Write the dual to the foregoing problem by associating a dual variable with each of the six inequality constraints.
f. Using the graph of Part (a), compute the set of dual optimal solutions and determine why or why not the dual has alternative optimal solu- tions.
g. Graphically add the constraint x_{1} + x_{2} <= 4 to the problem. Is there a degenerate optimal dual basic

Answers

The given bounded variables linear program has two decision variables, x₁ and x₂, and six inequality constraints. The objective is to maximize the expression x₁ + x₂. In this answer, we will solve the problem graphically, determine the optimal basic feasible partitions, perform one iteration of the simplex method for a specific extreme point, analyze the introduction of x₂ into the basis, derive the dual problem, compute the set of dual optimal solutions, and investigate the addition of a new constraint graphically.

a. To solve the problem graphically, we plot the feasible region determined by the given inequality constraints. The feasible region is bounded by the constraints -2x₁ + x₂ ≤ 2, x₁ - x₂ ≤ 0, -2 ≤ x₁ ≤ 2, and -1 ≤ x₂ ≤ 2. The objective function x₁ + x₂ represents a line with a positive slope in the (x₁, x₂) space. By examining the feasible region and evaluating the objective function at its extreme points, we can identify the optimal solution.

b. The optimal basic feasible partitions are determined by selecting subsets of the decision variables as basic variables, while the remaining variables are nonbasic. In this case, the sets of basic and nonbasic variables at optimality will depend on the extreme points of the feasible region and the objective function. By evaluating the objective function at each extreme point, we can identify the optimal partitions.

c. For the extreme point (0, 2), we construct the bounded variables simplex tableau. The tableau includes the coefficients of the decision variables and slack variables, as well as the corresponding values for the objective function and constraints. By performing one iteration of the simplex method, we update the tableau to improve the objective function value. Whether the resulting tableau is optimal or not depends on the optimality conditions.

d. To verify the statement graphically, we start at a specific point where the slack from the second constraint and x₂ are nonbasic at their lower bounds. By introducing x₂ into the basis, we move to a new basic feasible solution. Whether this new solution is optimal or not depends on the objective function and the feasibility of the solution. Graphically analyzing the feasible region can help determine if the resulting solution is indeed optimal.

e. To write the dual problem, we associate a dual variable with each of the six inequality constraints. Letting s₁, s₂, x₃, x₄, x₅, and x₆ represent the dual variables corresponding to the constraints, the dual problem involves minimizing a linear combination of the dual variables subject to dual constraints. The dual variables are associated with the inequality constraints in the opposite direction, and the objective of the dual problem is to minimize the expression -2s₁ + s₂ + 2x₃ + x₄ + 2x₅ + x₆.

f. By utilizing the graph from part (a), we can compute the set of dual optimal solutions. The dual optimal solutions correspond to the extreme points of the dual feasible region, which can be determined by graphically analyzing the relationship between the objective function of the dual problem and the dual constraints. The existence of alternative optimal solutions for the dual problem depends on the shape and properties of the primal feasible region.

g. Adding the constraint x₁ + x₂ ≤ 4 to the problem introduces a new boundary to the feasible region. By graphically analyzing the updated feasible region, we can determine if there is a degenerate optimal dual basic solution. The degeneracy of the solution depends on whether the new constraint intersects with the existing constraints, resulting in multiple optimal solutions for the dual problem.

To learn more about function click here: brainly.com/question/30721594

#SPJ11

Problem Three. Evaluate
∫∫Ώ (x + y)² dxdy
where isΏ the parallelogram bounded by the lines 2x + 3y = 1, 2x + 3y - 3 3x - 2y = 0, 3x - 2y = 4.

Answers

The parallelogram bounded by the lines 2x + 3y = 1, 2x + 3y - 3 3x - 2y = 0, 3x - 2y = 4,0 ≠ -4, there is no intersection point between these two lines.

The double integral ∫∫Ώ (x + y)² dxdy over the region Ώ, which is the parallelogram bounded by the lines 2x + 3y = 1, 2x + 3y - 3 = 0, 3x - 2y = 0, and 3x - 2y = 4,  to find the limits of integration for x and y.

To determine the limits of integration,  the intersection points of the given lines.

The intersection of the lines 2x + 3y = 1 and 2x + 3y - 3 = 0:

Subtracting the second equation from the first equation,

(2x + 3y) - (2x + 3y - 3) = 1 - 0

3 = 1

Since 3 ≠ 1, there is no intersection point between these two lines.

find the intersection of the lines 2x + 3y = 1 and 3x - 2y = 0:

Solving the system of equations,

2x + 3y = 1 ...(1)

3x - 2y = 0 ...(2)

Multiplying equation (1) by 3 and equation (2) by 2,

6x + 9y = 3 ...(3)

6x - 4y = 0 ...(4)

Subtracting equation (4) from equation (3),

(6x + 9y) - (6x - 4y) = 3 - 0

13y = 3

Simplifying,

y = 3/13

Substituting this value of y into equation (2),  solve for x:

3x - 2(3/13) = 0

3x = 6/13

x = 2/13

Therefore, the intersection point of the lines 2x + 3y = 1 and 3x - 2y = 0 is (x, y) = (2/13, 3/13).

the intersection of the lines 3x - 2y = 0 and 3x - 2y = 4:

Subtracting the second equation from the first equation,

(3x - 2y) - (3x - 2y) = 0 - 4

0 = -4

To know more about parallelogram  here

https://brainly.com/question/28854514

#SPJ4

The following data show the monthly salaries of a sample of IBM
graduates.
IBM Student
Monthly Salary (in 1,000s Rupees)
A. 78
B. 87
C. 80
D. 100
E. 104
F. 88

Answers

The median monthly salary of the IBM graduates in the given data set is 87.5 thousand rupees.

To find the median of the given data set, the first step is to arrange the given data set in ascending order.

The data set is:{78, 87, 80, 100, 104, 88}

After arranging the data set in ascending order, it becomes:{78, 80, 87, 88, 100, 104}

There are six data points in the given data set.

To find the median, the middle data point must be found. In this case, there are two middle data points because there are an even number of data points.

To find the median of the data set, the two middle data points must be averaged.

The two middle data points are 87 and 88.

To find the average of these two data points, add them together and divide by 2:

(87 + 88)/2 = 175/2 = 87.5

Therefore, the median monthly salary of the IBM graduates in the given data set is 87.5 thousand rupees.

Know more about median here:

https://brainly.com/question/26177250

#SPJ11

Find the probability of being dealt a holdem hand with two
hearts. What is the probability of flopping a flush given that you
have 2 hearts? (Express as % and round to 2 digits)

Answers

The probability of being dealt a holdem hand with two hearts is 10.44%. The probability of flopping a flush given that you have 2 hearts is 10.94%.

There are 52 cards in a deck. A holdem hand consists of 2 cards. Therefore, there are C(52, 2) possible holdem hands: \[{52 \choose 2}\] = (52 * 51) / (2 * 1) = 1326There are 13 hearts in a deck. The probability of getting one heart in your first card is 13/52.

Since there are 12 hearts remaining in the deck, the probability of getting another heart on your second card is 12/51.

So the probability of getting dealt a holdem hand with two hearts is: (13/52) * (12/51) = 0.0498, or 4.98%.

However, there are C(13, 2) possible combinations of two hearts in a deck: \[{13 \choose 2}\] = (13 * 12) / (2 * 1) = 78

So the probability of getting dealt a holdem hand with two hearts is 78/1326 = 0.1044, or 10.44%.If you have two hearts, there are 11 hearts left in the deck.

Therefore, the probability of flopping a flush is the number of ways to pick 3 hearts out of 11, divided by the number of ways to pick 3 cards out of 50 (the remaining cards in the deck).

This is given by: \[\frac{{{11 \choose 3}}}{{{50 \choose 3}}}\] = 0.1094, or 10.94%.

So the probability of flopping a flush given that you have 2 hearts is 10.94%.

To know more about probability visit :-

https://brainly.com/question/31828911

#SPJ11

A dice game involving rolling 2 dice pays 6 units if you roll a
total of 7, pays zero if you roll a 2 or 12, and you lose one unit
otherwise. Find the expected value and standard deviation of a unit
b

Answers

The probability distribution for rolling two dice is as follows:Roll 2: 1/36Roll 3: 2/36Roll 4: 3/36Roll 5: 4/36Roll 6: 5/36Roll 7: 6/36Roll 8: 5/36Roll 9: 4/36Roll 10: 3/36Roll 11: 2/36Roll 12: 1/

The formula for expected value is E(X) = Σ(x * P(x)), where x is the value of the outcome and P(x) is the probability of that outcome occurring.

Using the probability distribution from above, we can calculate the expected value:

Using the same probability distribution, we can calculate the standard deviation:

Standard deviation = ≈ 2.42 units

Summary: The expected value of rolling two dice in the described game is 0.5 units, while the standard deviation is approximately 2.42 units.

Learn more about probability click here:

https://brainly.com/question/13604758

#SPJ11

A matrix and a scalar λ are given. Show that λ is an eigenvalue of the matrix and determine a basis for its eigenspace
[6 9 -10], λ = 5
[6 3 -4 ]
[7 7 -9 ]

Answers

To determine if λ = 5 is an eigenvalue of the given matrix, we need to find a non-zero vector v such that Av = λv, where A is the given matrix.

Let's set up the equation: A - λI = [6-5 9 -10] = [1 9 -10]. [6 3 -4 ] [6 -2 -4 ]

[7 7 -9 ] [7 7 -14]. To find the eigenvector v, we need to solve the equation (A - λI)v = 0. Setting up the augmented matrix:[1 9 -10 | 0]. [6 -2 -4 | 0]. [7 7 -14 | 0] Performing row reduction operations: R2 - 6R1 -> R2. R3 - 7R1 -> R3 . [1 9 -10 | 0].  [0 -56 56 | 0]. [0 -56 56 | 0]. R2 / (-56) -> R2. R3 - R2 -> R3. [1 9 -10 | 0]. [0 1 -1 | 0]. [0 0 0 | 0]. From the row-reduced form, we can see that the matrix has a free variable. Let's choose a value for the free variable, say t = 1, and solve for the other variables: x + 9y - 10z = 0 --> x = -9y + 10z. y - z = 0 --> y = z. Using the parameter z, we can express the eigenvector v: v = [-9y + 10z, y, z] = [-9y + 10z, y, z]. Choosing y = 1 and z = 1, we get: v = [-9(1) + 10(1), 1, 1] = [1, 1, 1]. Thus, the eigenvector corresponding to the eigenvalue λ = 5 is v = [1, 1, 1].

To find the basis for the eigenspace, we can multiply the eigenvector by any scalar. Therefore, a basis for the eigenspace is {k[1, 1, 1]}, where k is a non-zero scalar.

To learn more about eigenvalue   click here: brainly.com/question/32575123

#SPJ11

Put the steps in order to produce the output shown below. Assume the indenting will be correct in the program.
1 3
5 3
1 7
5 7

Answers

To produce the output "1 35 31 75 7" with correct indenting in a program, the steps are as follows: 1, 31, 35, 7, 75.

To generate the output "1 35 31 75 7" with correct indenting in a program, we need to arrange the steps in the correct order. Let's analyze the given output:

1 35 31 75 7

From this output, we can deduce that the numbers are arranged in ascending order. The correct order of the steps to produce this output is as follows:

Start with the smallest number, which is 1.

Move to the next smallest number, which is 31.

Proceed to the next number, which is 35.

Continue to the second-largest number, which is 75.

Finally, include the largest number, which is 7.

By following these steps in order, and with correct indenting in the program, we will obtain the desired output: "1 35 31 75 7".

Learn more about ascending order here:

https://brainly.com/question/24423536

#SPJ11

Let f(x)=√x + 2. Calculate the difference quotient
f(47+h)-f(47)/ h for

h = .1
h = .01
h = -.01
h = -.1

If someone now told you that the derivative (slope of the tangent line to the graph) of
f(x) at 1 47 was for some integer n what would you expect n to be?
I= .01 .1
n= n

Answers

The difference quotient Hence we can choose n = 0.I = 0.01, 0.1n = 0

Given that f(x)=√x + 2.

The formula for the difference quotient is

f(x) = (f(x + h) - f(x))/h

For f(x)=√x + 2f(x + h) = √(x+h) + 2

Thus the difference quotient is given by(f(x + h) - f(x))/h = [√(x+h) + 2 - √x - 2]/h

Simplify the expression above(f(x + h) - f(x))/h = [√(x+h) - √x]/h

After multiplying by the conjugate of the numerator, we get,

(f(x + h) - f(x))/h = [(√(x+h) - √x)/(h)] × [√(x+h) + √x)/(√(x+h) + √x)](f(x + h) - f(x))/h

= [√(x+h) - √x]/[(x+h) - x] × [√(x+h) + √x)]/(√(x+h) + √x)](f(x + h) - f(x))/h = [√(x+h) - √x]/[h×(√(x+h) + √x)]

For h = 0.1,f(47 + 0.1) = √(47 + 0.1) + 2 = 9.87517f(47) = √47 + 2 = 9.08276(f(47 + 0.1) - f(47))/0.1 = (9.87517 - 9.08276)/0.1 = 7.92614

For h = 0.01,f(47 + 0.01) = √(47 + 0.01) + 2 = 9.48723f(47) = √47 + 2 = 9.08276(f(47 + 0.01) - f(47))/0.01 = (9.48723 - 9.08276)/0.01 = 40.1238

For h = -0.01,f(47 - 0.01) = √(47 - 0.01) + 2 = 9.4748f(47) = √47 + 2 = 9.08276(f(47 - 0.01) - f(47))/(-0.01) = (9.4748 - 9.08276)/(-0.01) = -39.2324

For h = -0.1,f(47 - 0.1) = √(47 - 0.1) + 2 = 9.86802f(47) = √47 + 2 = 9.08276(f(47 - 0.1) - f(47))/(-0.1) = (9.86802 - 9.08276)/(-0.1) = -7.8526

Given that the derivative (slope of the tangent line to the graph) of f(x) at 47 was for some integer n.

We have to find the value of n such that

f'(47) = n

where

f'(x) = (d/dx)√x + 2f'(x) = 1/(2√x + 4)f'(47) = 1/(2√47 + 4)f'(47) ≈ 0.08845

Now we need to find an integer that is close to 0.08845.

Hence we can choose n = 0.I = 0.01, 0.1n = 0

To know more about derivative visit:

https://brainly.com/question/29144258

#SPJ11

In (r, q) coordinates A = (83.0, 344 degrees) and B = (69.0, 2.90E2 degrees). Given R = = A - B A - (a) In polar coordinates the resultant vector is R = (₁ Rr, Re Ro). What is the radial component, Rr?

Answers

To find the radial component, Rr, of the resultant vector R in polar coordinates, we need to subtract the radial components of the vectors A and B. Rr represents the magnitude of the radial displacement in the polar coordinate system.

In polar coordinates, a vector is represented by its radial distance from the origin (Rr) and its angle from the positive x-axis (Re). We are given the coordinates of vectors A and B in (r, q) form.

Vector A is given as A = (83.0, 344 degrees) and vector B is given as B = (69.0, 290 degrees).

To find the resultant vector R = A - B, we subtract the radial components and add the angular components.

Rr = |RrA - RrB|

= |83.0 - 69.0|

= |14.0|

= 14.0

The radial component, Rr, of the resultant vector R is 14.0 in the given polar coordinate system. It represents the magnitude of the radial displacement or distance from the origin.

Learn more about Vector here:

https://brainly.com/question/24256726

#SPJ11

five students are asked to randomly select and name a polygon from the group shown below. what is the probability that the first two students choose the triangle and the rectangle?

Answers

To find the probability that the first two students choose the triangle and the rectangle, consider the total number of polygons available and the number of favorable outcomes which will be (1/n) * (1/(n-1)).

Assuming all polygons in the group are equally likely to be chosen, let's consider the total number of polygons available. From the given information, we do not know the exact number of polygons in the group.

Let's denote the total number of polygons as 'n'. The first student has a probability of 1/n to choose the triangle, and after the triangle is chosen, the second student has a probability of 1/(n-1) to choose the rectangle, as there is one less polygon remaining.

Therefore, the probability that the first two students choose the triangle and the rectangle is (1/n) * (1/(n-1)). The exact value of this probability depends on the total number of polygons 'n' in the group.

Learn more about probability here: brainly.com/question/473039
#SPJ11

"Set up, but do not evaluate, an integral for the volume of the solid obtained by rotating the region bounded by y = 0, y = sin(x), and 0 < x < π about the line y = -2. Please also provide a sketch of the region and the line of rotation."

Answers

The integral for the volume generated is V = ∫[0, π] 2π(x + 2) [sin(x)] dx

How to set up the integral for the volume generated

From the question, we have the following parameters that can be used in our computation:

y = 0 and y = sin(x)

Also, we have

The line u = -2

Set the equations to each other

So, we have

sin(x) = 0

When evaluated, we have

x = 0 and x = π

For the volume generated from the rotation around the region bounded by the curves, we have

V = ∫[a, b] 2π(x + 2) [g(x) - f(x)] dx

This gives

V = ∫[0, π] 2π(x + 2) [sin(x) - 0] dx

So, we have

V = ∫[0, π] 2π(x + 2) [sin(x)] dx

Hence, the integral for the volume generated is V = ∫[0, π] 2π(x + 2) [sin(x)] dx

Read more about volume at

brainly.com/question/11942113

#SPJ1

Determine the vector and parametric equations of the line going through the points P(1,2,4) and Q(1,3,6). Question 17 (3 points) Do the lines L1​:r=(1,7,−5)+s(2,−2,5),s∈R, and the line L2​:r=(−2,3,−6)+s(3,2,6),s∈R, determine a plane?

Answers

The equation of the plane is:r = (1, 7, −5) + s(2, −2, 5) + t(3, 2, 6)

Where s, t ∈ R.

Solution: The vector and parametric equations of the line going through the points P(1, 2, 4) and Q(1, 3, 6) are given below: Vector Equation :We will determine the direction vector by subtracting the coordinates of two points Q and P.

r = OP + t PQ= (1, 2, 4) + t (0, 1, 2)

Here, OP is the position vector of P, and PQ is the vector from P to Q.

The direction vector of the line L is PQ (0, 1, 2).Parametric Equation:

Now we will express the vector equation in parametric form.

x = 1 + 0ty = 2 + t, and z = 4 + 2

t where t ∈ R.  the lines L1​: r = (1, 7, −5) + s(2, −2, 5), s ∈ R, and

the line L2​: r = (−2, 3, −6) + s(3, 2, 6), s ∈ R, determine a plane.

Let us find two points that lie on both of these lines to find the plane of intersection:

Let point A lie on line L1, such that A = (1, 7, −5)Let point B lie on line L2, such that B = (−2, 3, −6)

Equation of line L1 is given as:r1 = (1, 7, −5) + s(2, −2, 5)

Let's find two values of s such that r1 lies on line L2:r1 = (1, 7, −5) + s(2, −2, 5)= (1 + 2s, 7 − 2s, −5 + 5s)

Now we can equate the two vectors r1 and r2:r1 = r2⟹(1 + 2s, 7 − 2s, −5 + 5s) = (−2 + 3t, 3 + 2t, −6 + 6t)From this system of equations,

we can determine the values of s and t such that the two points coincide and lie on both lines.

Now we solve the system of equations:1 + 2s = −2 + 3t7 − 2s = 3 + 2t−5 + 5s = −6 + 6tSolving the system,

we get: s = −1 and t = 1

We can check if the points A and B lie on both lines:L1, s = −1: r1 = (−1, 9, 0)L2, t = 1: r2 = (1, 5, 0)

We can see that the two points A and B both lie on the plane with the equation: r = r0 + s v1 + t v2

Where r0 is the position vector of A, and v1, v2 are the direction vectors of the lines L1 and L2, respectively.

Substituting the values:r0 = (1, 7, −5)v1 = (2, −2, 5)v2

= (3, 2, 6)

To know more about equation visit:

https://brainly.com/question/29538993

#SPJ11

If x, y, z be in HP prove that (y+x)/(y-x)+(y+z)/(y-z) = 2 ​

Answers

If x, y, and z be in Harmonic progression, then the equation (y+x)/(y-x)+(y+z)/(y-z) = 2 ​is satisfied.

The reciprocal of Harmonic progression (HP) is arithmetic progression (AP),

Let d be a common difference,

1/x, 1/y, and 1/z are in AP.

1/y - 1/x = d

1/z - 1/y = d

where d is the common difference,

Evaluating equations.

(y+x)/(y-x) + (y+z)/(y-z)

[(y+x)(y-z) + (y+z)(y-x)] / [(y-x)(y-z)]

[2y² - 2xz] / [(y-x)(y-z)]

Substituting value of d,

[2y² - 2xz] / [(-d)(d)]

[2y² - 2xz] / (d²) = 2

By solving, we get

y² - xz = d²

The common difference in the AP is equal to the difference between two successive terms.

Therefore, d² = xz and d² = y²

y² - xz = xz

y² = 2xz

= 2

Hence, (y+x)/(y-x)+(y+z)/(y-z) = 2.

To learn more about Harmonic progression, here:

https://brainly.com/question/27927304

#SPJ1


IF I SPEND $6,300 OUT OF $21,000 WHAT PERCENT DID I SPEND

Answers

Answer: 30%

Step-by-step explanation: Solution for 6300 is what percent of 21000: 6300:21000*100 = (6300*100):21000 = 630000:21000 = 30. Now we have: 6300 is what percent of 21000 = 30.

if we take 21000(origin amount) to be the 100%, what's 6300 off of it in percentage?

[tex]\begin{array}{ccll} Amount&\%\\ \cline{1-2} 21000 & 100\\ 6300& x \end{array} \implies \cfrac{21000}{6300}~~=~~\cfrac{100}{x} \\\\\\ \cfrac{10}{3} ~~=~~ \cfrac{100}{x}\implies 10x=300\implies x=\cfrac{300}{10}\implies x=30[/tex]

what correctly displays a realationship between sets of real numbers

Answers

A relationship between sets of real numbers can be accurately represented through mathematical concepts such as subsets, intersections, unions, and equalities.

When comparing sets of real numbers, various mathematical concepts help express the relationship between them. One fundamental concept is the subset. A set A is considered a subset of another set B if every element in A is also an element in B. This relationship is denoted as A ⊆ B. For example, if A = {1, 2} and B = {1, 2, 3}, then A is a subset of B since all the elements in A are also present in B.

Another useful concept is the intersection of sets. The intersection of sets A and B, denoted as A ∩ B, refers to the set of elements that are common to both sets. For instance, if A = {1, 2, 3} and B = {2, 3, 4}, the intersection of A and B would be {2, 3} since those are the elements shared by both sets.

Furthermore, the union of sets provides a way to combine elements from multiple sets. The union of sets A and B, denoted as A ∪ B, represents the set that contains all the elements from both sets without duplication. For example, if A = {1, 2, 3} and B = {3, 4, 5}, the union of A and B would be {1, 2, 3, 4, 5}.

Lastly, the concept of equality between sets implies that two sets have exactly the same elements. If all the elements of set A are present in set B, and vice versa, then A = B. However, it's important to note that the order of elements within a set is irrelevant for equality.

By utilizing these mathematical concepts, one can accurately represent and analyze the relationship between sets of real numbers.

Learn more about subsets here:

https://brainly.com/question/28705656

#SPJ11

for what value of a would the following system of equations have an infinite number of solutions?
2x - y = 8
6x - 3y = 41

A. 2
B. 6
C. 8
D. 24
E. 32

Answers

Therefore, the value of "a" that would result in an infinite number of solutions is a = 2 that is option A.

To determine the value of "a" that would result in an infinite number of solutions for the system of equations, we need to check if the two equations are proportional or equivalent to each other.

Let's manipulate the second equation by dividing both sides by 3:

2x - y = 8

2x - (1/3)y = 41/3

Now, if we multiply the second equation by a, we can compare it to the first equation:

2x - (1/3)y = 41/3

a(2x - (1/3)y) = a(8)

Simplifying both sides:

2ax - (a/3)y = 8a

We can see that if "a" is equal to 3, the two equations become identical:

2(3)x - (3/3)y = 8(3)

6x - y = 24

To know more about solutions,

https://brainly.com/question/14633199

#SPJ11

what is the equation of a line that passes through the point (2, −10) and is parallel to 14x 2y=6?

Answers

The equation of line that passes through the point (2, -10) and is parallel to 14x - 2y = 6 is y = -3.5x - 3.

A line parallel to 14x - 2y = 6 will have the same slope as the given line, which can be found by rearranging the equation into slope-intercept form:

14x - 2y = 6-2y = -14x + 6y = 7x - 3y = -3.5x + 1.5

The slope of this line is -3.5,

so the slope of any parallel line will also be -3.5.

We also know that this line passes through the point (2, -10).

Using point-slope form, the equation of the line is:y - y1 = m(x - x1), where m is the slope and (x1, y1) is the given point.

y - (-10) = -3.5(x - 2)y + 10 = -3.5x + 7y = -3.5x - 3

Let's verify that this equation represents a line parallel to the given line:

14x - 2y = 6-2y = -14x + 6y = 7x - 3y = -3.5x + 1.5

The slopes of both lines are -3.5, so they are parallel.

Therefore, the equation of a line that passes through the point (2, -10) and is parallel to 14x - 2y = 6 is y = -3.5x - 3.

Know more about the slope-intercept form

https://brainly.com/question/1884491

#SPJ11

The inverse of a diagonal matrix is a diag- onal matrix with each element inverted. C 0 0 C2 In other words, if A = ... Сп 0 (1/4 0 then A-1 = 1/C2 1/c, (a) Prove this fact mathematically. (b) Now explain it verbally by saying what effect A and A-1 have as trans- formations of an n-dimensional vector.

Answers

The inverse of a diagonal matrix is obtained by taking the reciprocal of each diagonal element, resulting in a diagonal matrix with inverted values.

(a) To prove this fact mathematically, let A be a diagonal matrix with diagonal elements C1, C2, ..., Cn. The inverse of A, denoted as A-1, can be found by taking the reciprocal of each diagonal element. Therefore, the diagonal elements of A-1 are 1/C1, 1/C2, ..., 1/Cn. Since both A and A-1 are diagonal matrices with the same dimensions, this proves that the inverse of a diagonal matrix is a diagonal matrix with each element inverted.

(b) Geometrically, a diagonal matrix represents a scaling transformation along the coordinate axes. Each diagonal element Ci scales the corresponding coordinate by a factor of Ci. When we take the inverse of a diagonal matrix, A-1, it effectively reverses the scaling by inverting each scaling factor. Therefore, multiplying a vector by A results in scaling its coordinates by Ci, while multiplying the same vector by A-1 scales the coordinates by 1/Ci. In other words, A stretches or shrinks the vector along the coordinate axes, while A-1 performs the opposite scaling, compressing or elongating the vector along the coordinate axes.

Learn more about  matrix here:

https://brainly.com/question/29132693

#SPJ11

Final answer:

The word created diagonally from left to right is FORT.

Explanation:

To find the word created diagonally from left to right, we need to examine the given words: FORM, COMA, FORD, and TALK. By looking at these words, we can see that the letters 'F', 'O', 'R', and 'T' are aligned diagonally from left to right. Therefore, the word created diagonally is FORT.

Learn more about diagonal word formation here:

https://brainly.com/question/32735590

#SPJ14

In your answers below, for the variable λ type the word lambda; for the derivative ddxX(x) type X' ; for the double derivative d2dx2X(x) type X''; etc. Separate variables in the following partial differential equation for u(x,t): t2uxx+x2uxt−x2ut=0

Answers

The given partial differential equation is t^2u_xx + x^2u_xt - x^2u_t = 0. In this equation, u(x,t) represents the unknown function of two variables, x and t.

To express the equation in a standardized notation, we replace the partial derivatives with their respective symbols: u_xx represents the second partial derivative of u with respect to x, u_xt represents the mixed partial derivative of u with respect to x and t, and u_t represents the partial derivative of u with respect to t.

The equation can be rewritten as t^2u_xx + x^2u_xt - x^2u_t = 0. This form highlights the differentiating variables and their coefficients. It represents a partial differential equation involving second-order derivatives with respect to x and first-order derivatives with respect to t.

To solve this partial differential equation, various methods such as separation of variables, method of characteristics, or numerical methods can be employed, depending on the specific problem and boundary conditions.

To know more about partial differential equations click here: brainly.com/question/30226743

#SPJ11

Representing a large auto dealer, a buyer attends car auctions. To help with the bidding, the buyer built a regression equation to predict the resale value of cars purchased at the auction. The equation is given below. Estimated Resale Price ($) = 20,000 - 2,050 Age (year), with p = 0.52 and se = $3,200 = Use this information to complete parts (a) through (c) below. (a) Which is more predictable: the resale value of one six-year-old car, or the average resale value of a collection of 16 cars, all of which are six years old? A. The resale value of one six-year-old car is more predictable because only one car will contribute to the error. B. The average of the 16 cars is more predictable by default because it is impossible to predict the value of a single observation. C. The average of the 16 cars is more predictable because the averages have less variation. D. The resale value of one six-year-old car is more predictable because a single observation has no variation. (b) According the buyer's equation, what is the estimated resale value of a six-year-old car? The average resale value of a collection of 16 cars, each six years old? The estimated resale value of a six-year-old car is $ (Type an integer or a decimal. Do not round.) The average resale value of a collection of 16 cars, each six years old is $ (Type an integer or a decimal. Do not round.) (c) Could the prediction from this equation overestimate or underestimate the resale price of a car by more than $2,250? O A. No. Since $2,250 is less than the standard error of $3,200, it is impossible for the regression equation to be off by more than $2,250. B. No. Since $2,250 is greater than the absolute value of the predicted slope, $2,050, it is impossible for the regression equation to be off by more than $2,250. C. Yes. Since $2,250 is less than the standard error of $3,200, it is quite possible that the regression equation will be off by more than $2,250. D. Yes. Since $2,250 is greater than the absolute value of the predicted slope, $2,050, it is quite possible that the regression equation will be off by more than $2,250.

Answers

The estimated resale value of a six-year-old car is $12,200. The prediction from this equation could potentially overestimate or underestimate the resale price of a car by more than $2,250.

(a) The average resale value of a collection of 16 six-year-old cars is more predictable than the resale value of one individual six-year-old car. This is because the average of multiple observations tends to have less variation and is more representative of the overall trend. When taking an average, the individual variations tend to cancel out, resulting in a more reliable estimate.

(b) According to the buyer's equation, the estimated resale value of a six-year-old car is $12,200. The average resale value of a collection of 16 six-year-old cars would be the same, $12,200, since the equation gives a fixed value for each six-year-old car.

(c) Yes, the prediction from this equation could potentially overestimate or underestimate the resale price of a car by more than $2,250. The standard error of the estimate (se) is $3,200, which indicates the typical amount of variation in the predicted values. Since $2,250 is less than the standard error, it is possible for the regression equation to be off by more than $2,250. The absolute value of the predicted slope ($2,050) is not directly related to the potential overestimation or underestimation. The standard error provides a more appropriate measure of the potential variability in the predictions.

Learn more about equation here:

https://brainly.com/question/28243079

#SPJ11

Determine whether each sequence is arithmetic, geometric, or neither. If it's arithmetic, make sure to choose the correct value of the common difference d. If it's geometric, make sure to choose the correct value of the common ratio r. A.) an = -2, -4,-8,-16,... B.) an = -4,-2,0, 2, 4,... C.) an = -4n D.) an = n⁻⁴

Answers

A.) The sequence is geometric with a common ratio of r = -2. B.) The sequence is arithmetic with a common difference of d = 2. C.) The sequence is arithmetic with a common difference of d = -4. D.) The sequence is neither arithmetic nor geometric.

A.) The given sequence -2, -4, -8, -16,... is a geometric sequence because each term is obtained by multiplying the previous term by -2. The common ratio is -2.

B.) The sequence -4, -2, 0, 2, 4,... is an arithmetic sequence because each term is obtained by adding 2 to the previous term. The common difference is 2.

C.) The sequence -4n is an arithmetic sequence because each term is obtained by subtracting 4 from the previous term. The common difference is -4.

D.) The sequence an = n⁻⁴ is neither arithmetic nor geometric. It is a power sequence with each term obtained by raising n to the power of -4. There is no constant ratio or difference between terms.

In conclusion, sequence A is geometric with a common ratio of -2, sequence B is arithmetic with a common difference of 2, sequence C is arithmetic with a common difference of -4, and sequence D is neither arithmetic nor geometric.

learn more about geometric sequence here: brainly.com/question/27852674

#SPJ11

Which matrix represents reflection about the ry-plane?

Answers

The matrix that represents reflection about the yz-plane, also known as the ry-plane, is:

[ -1  0  0 ]
[  0  1  0 ]
[  0  0 -1 ]

To understand the matrix that represents reflection about the yz-plane (ry-plane), we need to consider the coordinate system. In a three-dimensional Cartesian coordinate system, the yz-plane is a plane that lies parallel to the x-axis. Reflection about this plane involves flipping the sign of the x-coordinate while leaving the y and z coordinates unchanged.
The matrix representation of this reflection operation can be obtained by considering the effect it has on the standard basis vectors. The standard basis vectors are the vectors that have a single component equal to 1, and all other components equal to 0. In this case, we consider the basis vectors i, j, and k, which represent the unit vectors along the x, y, and z axes, respectively.
When the reflection operation is applied to these basis vectors, the resulting vectors are:i -> -i
j -> j
k -> -k
By arranging these resulting vectors as columns of a matrix, we obtain the reflection matrix for the yz-plane:[ -1  0  0 ]
[  0  1  0 ]
[  0  0 -1 ]
Therefore, this matrix represents reflection about the yz-plane or ry-plane in a three-dimensional Cartesian coordinate system.

learn.more about matrix here

https://brainly.com/question/29132693



#SPJ11

(Discrete mathematics), please help will upvote thanks! Please show step-by-step!

Consider the function f : P(Z) → P(Z) defined by f(X) = X.

a) Prove that f is a function.

b) Prove that f is onto.

c) Prove that f is one-to-one.

Answers

a) To prove that f : P(Z) → P(Z) is a function, we need to show that for every input set X in the power set of Z, there exists a unique output set Y in the power set of Z.

Let's consider an arbitrary input set X in the power set of Z. Since X is in the power set of Z, it means that X is a subset of Z.

Now, let's apply the function f to X, which is defined as f(X) = X. Since the function simply maps the input set to itself, there is no ambiguity or multiple outputs possible. For any given input set X, the output set Y = X, which is a subset of Z.

Therefore, for every input set X in the power set of Z, there exists a unique output set Y = X. This confirms that f is a function.

b) To prove that f : P(Z) → P(Z) is onto, we need to show that for every set Y in the power set of Z, there exists an input set X in the power set of Z such that f(X) = Y.

Consider an arbitrary set Y in the power set of Z. Since Y is in the power set of Z, it means that Y is a subset of Z.

Now, let's find the input set X that satisfies f(X) = Y. Since f(X) = X, we need to find a set X such that X = Y.

It is clear that if we choose X = Y, then f(X) = f(Y) = Y, which satisfies the condition.

Therefore, for every set Y in the power set of Z, we can find an input set X such that f(X) = Y. This shows that f is onto.

c) To prove that f : P(Z) → P(Z) is one-to-one, we need to show that for any two distinct input sets X and X' in the power set of Z, their corresponding output sets f(X) and f(X') are also distinct.

Let X and X' be two distinct sets in the power set of Z. Since X and X' are distinct, there must exist at least one element that belongs to one set but not the other.

Without loss of generality, let's assume there exists an element a such that a is in X but not in X'. Mathematically, a ∈ X and a ∉ X'.

Now, let's consider the corresponding output sets f(X) and f(X'). Since f(X) = X and f(X') = X', we have: f(X) = X, f(X') = X'

From the assumption that a is in X but not in X', we can see that a is an element of f(X) but not of f(X'). Mathematically, a ∈ f(X) and a ∉ f(X').

This proves that f(X) and f(X') are distinct output sets.

Therefore, for any two distinct input sets X and X' in the power set of Z, their corresponding output sets f(X) and f(X') are also distinct. This confirms that f is one-to-one.

LEARN MORE ABOUT function here: rainly.com/question/31062578

#SPJ11

On average, college seniors graduating in 2012 could compute their growing student loan debt using the function D(t) = 29,400(1.068)', where t is time in years. Which expression is equivalent to 29,40

Answers

The expression equivalent to 29,400(1.068)' and can be used to identify an approximate daily interest rate on the loans is option 1: 29,400 * 1.068.

In the given function D(t) = 29,400(1.068)', the term (1.068)' represents the growth factor over time, which is calculated as 1.068 raised to the power of 't'. This factor accounts for the compounding effect of the interest on the student loan debt.

To identify an approximate daily interest rate, we need to isolate the factor that corresponds to the daily rate within the function. Since 365 days make up a year, dividing the annual growth factor (1.068) by 365 will give us an approximate daily interest rate.

Therefore, the expression 29,400 * 1.068 represents the initial loan amount multiplied by the annual growth factor. By dividing this expression by 365, we can estimate the daily interest rate on the loans. Therefore, Option 1 is correct.

The question was incomplete. find the full content below:

On average, college seniors graduating in 2012 could compute their growing student loan debt using the function D(t) = 29,400(1.068)', where t is time in years. Which expression is equivalent to 29,400(1.068)' and could be used by students to identify an approximate daily interest rate on their loans? 365 1) 29,400 1.068 1.068  2) 29,400 365  3) 29,400 1+ 29,4001  4) 29,400 1.068 365t 0.068 365 365 365t

Know more about interest rate here:

https://brainly.com/question/29451175

#SPJ8

For the sample: 2, 21, 45, 45, 35, 22, 17, 19, 12, 22, 7; the range is:

Answers

In the given sample of numbers: 2, 21, 45, 45, 35, 22, 17, 19, 12, 22, 7, the range is 43.

The range is a statistical measure that indicates the spread or dispersion of a set of data. To calculate the range, we find the difference between the maximum and minimum values in the sample.Looking at the given sample, the minimum value is 2 and the maximum value is 45. To find the range, we subtract the minimum value from the maximum value:

Range = Maximum value - Minimum value

Range = 45 - 2 = 43.Therefore, the range of the sample is 43. This means that the values in the sample range from a minimum of 2 to a maximum of 45, with a difference of 43 between them. The range provides a simple measure of the spread of the data, giving us an idea of how spread out the values are in the sample.

Learn more about sample here:

https://brainly.com/question/27860316

#SPJ11

1. y=logb(2x-6) and b>1 what would the domain be in set builder notation?
2. what would be the y-intercept of this graph: f(n)=a^n + b where a is not equal to 1 and a > 0

Answers

The domain of the function y = log_b(2x – 6), where b > 1, is {x | x > 3}.
The y-intercept of the function f(n) = a^n + b, where a is not equal to 1 and a > 0, is the point (0, b).

The domain of the logarithmic function y = log_b(2x – 6), where b > 1, refers to the set of all valid input values for x. In this case, we need to ensure that the argument of the logarithm, 2x – 6, is greater than zero.

This is because the logarithm function is only defined for positive values.
To determine the domain, we solve the inequality 2x – 6 > 0:
2x – 6 > 0
2x > 6
X > 3

Therefore, the domain is expressed in set-builder notation as {x | x > 3}, meaning all values of x greater than 3.

The y-intercept of the function f(n) = a^n + b, where a is not equal to 1 and a > 0, is the point where the function intersects the y-axis, or when n = 0.

To find the y-intercept, we substitute n = 0 into the function:
F(0) = a^0 + b = 1 + b = b

Therefore, the y-intercept of the graph is (0, b), indicating that the y-coordinate is equal to the constant term b.


Learn more about logarithmic function here : brainly.com/question/30339782

#SPJ11



Other Questions
All other factors remaining equal, a firm's price/earnings ratio would decrease if there is, a. an equal decrease in price per share and earnings per share b. a decrease in investor risk aversion c. an increase in growth opportunities d. a decrease in net profit margin Solve the inequality. Write the solution set in interval notation and graph it. x-3x - 10> 0 a restaurant gives a discount for children under 10. they also give the discount for adults over 55. which expression evaluates to true if a discount should be given?a.(age < 10) The final, integrative individual assignment will be for you to prepare a list of the top 10 lessons for digital marketers that youve taken away from this course. What are the top 10 things you believe all digital marketers need to understand about consumer behavior to be successful with their strategy? There are 2 deliverables for this assignment a PowerPoint presentation and an infographic.You will have to pick specific concepts and ideas. Just going by chapter titles, headlines, captions, etc. will not suffice!! The "lesson" needs to be a unique point from the readings, videos, and your own experience. Note that strong presentations will use the ideas, theories, and research discussed throughout the course to frame the analysis and provide examples.Be sure to specify why you think each point you list deserves to be in the Top 10. What are the implications for digital marketers of each point? Explain why and how knowing this theory/fact concept, etc. will benefit digital marketers. Outline one or two examples of the point being used or implemented in a campaign. From a consumer behavior perspective, what is the firm doing right? What are some lessons to take away from this?Along with the slide deck, create an infographic (Canva is the best) to express your top 10 using all the elements of good design. This will take some time and has a lot of considerations when putting it togethe What is the name of the album that is most frequently cited as the beginning of fusion? Social exchange theory posits that relationship satisfaction depends on our perceptions of the rewards and costs associated with the relationship, what kind of relationship we believe we deserve, and whetherwe believe that a relationship with someone else would be better. suppose the government imposes an excise tax on a good. which of the following statements is true about the tax incidence on buyers and sellers of the good? The first order maximum caused by a double slit illuminated with light of wavelength 625 nm is found at some spot on a screen. The light source is changed to a new wavelength which places its second order (m=2) maxima at the same spot where the 625 nm first order maxima used to lie.(a) What is the wavelength of the new light source?(b) Is this wavelength is the visible range? The normal selling price is $20.00 per unit. The companyscapacity is 118,800 units per year. An order has been received froma mail-order house for 2,300 units at a special price of $17.00 peruniDelta Company produces a single product. The cost of producing and selling a single unit of this product at the company's normal activity level of 91,200 units per year is: Direct materials. $ 1.90 $ If the above entries are not recorded on November 30, 2020, what is the effect on the income statement and the balance sheet? (Enter all value as positive values. Round intermediate and the final answers to 2 decimal places. Select "No effect" in case of no effect on Income statement and Balance sheet.) Effect Amount Description The income statement would be On the balance sheet, liabilities would be On the balance sheet equity would be On the balance sheet assets would be Analysis Component: If the above entries are not recorded on November 30, 2020, what is the effect on the income statement and the balance sheet? (Enter all value as positive values. Round intermediate and the final answers to 2 decimal places. Select "No effect" in case of no effect on Income statement and Balance sheet.) Description The income statement would be Effect Amount On the balance sheet, liabilities would be On the balance sheet, equity would be On the balance sheet, assets would be Analysis Component: If the above entries are not recorded on November 30, 2020, what is the effect on the income statement and the balance sheet? (Enter all value as positive values. Round intermediate and the final answers to 2 decimal places. Select "No effect" in case of no effect on Income statement and Balance sheet.) Description The income statement would be Effect Amount On the balance sheet, liabilities would be On the balance sheet, equity would be On the balance sheet, assets would be Analysis Component: If the above entries are not recorded on November 30, 2020, what is the effect on the income statement and the balance sheet? (Enter all value as positive values. Round intermediate and the final answers to 2 decimal places. Select "No effect" in case of no effect on Income statement and Balance sheet.) Description The income statement would be Effect Amount On the balance sheet, liabilities would be On the balance sheet, equity would be On the balance sheet, assets would be Describe three types of evidence or data which have been used to determine themechanism(s) and rates of groundwater recharge in the Western Port Basin (southeastof Melbourne) (6 marks)b) Explain the major change in groundwater recharge mechanism that has occurred due tourban development in southeast Melbourne, explaining two types of evidence whichindicate this change Describe a performance review template for training ina luxury hotel. Given a relatively steep and a relatively flat short-run aggregate supply curve, which curve would support the case for nonactivist policy? Why? Identify commodity risk, currency risk and credit risk Section II (30%) (c) But only hedge 2 risks which are commodity risk and credit risk To make recommendations on which derivative instruments (for example, options, futures, etc.) to use to implement any hedges that you have recommended in part (b) above. Once again, you MUST explain your recommendations (i.e. justify the choice of the derivative instrument). of. Charles 5. Given that sin(x) = -1/2 and cos(y) = -2/5, x and y are in quadrant III, find: a. sin(x+y) b. cos(x+y) c. the quadrant of angle x+y 7. (Set up an integral, but do not evaluate.) Let R be the region bounded by the curves y = sin (x) for 0 x , and y = 0 (pictured below). Use the disk method to set up an integral that gives freedom from prosecution for witnesses whose testimony ties them to illegal acts is called An investment adviser is under the fiduciary responsibility to disclose all material facts in any: A loaded grocery cart is rolling across a parking lot in a strong wind. You apply a constant force F=(25 N)i-(45 N) to the cart as it undergoes a displacement 7=(-8.8 m)i (3.9 m). Part A How much work Engineers in an electric power company observed that they faced an average of 986 issues per month. Assume the standard deviation is 8. A random sample of 36 months was chosen. Find the 95% confidence interval of population mean.